Which equation has the same solution as x2 + 16x + 9 = -3?

Answers

Answer 1

Answer:

Step-by-step explanation:

x² + 16x + 9 = - 3

x² + 16x + 12 = 0

D = 16² - 4(1)(12) = 208

√D = 4√13

[tex]x_{1}[/tex] = ( - 16 - 4√13 ) ÷ 2 = - 8 - 2√13 ≈ - 15.21

[tex]x_{2}[/tex] = ( - 16 + 4√13 ) ÷ 2 = 2√13 - 8 ≈ - 0.79


Related Questions

Preparing a glucose solution In a certain medical test
designed to measure carbohydrate tolerance, an adult
drinks 7 ounces of a 30% glucose solution. When the test is
administered to a child, the glucose concentration must be
decreased to 20%. How much 30% glucose solution and
how much water should be used to prepare 7 ounces of
20% glucose solution?

Answers

3.5 ounces hope this helps

the degree of the polynomial -5 + 3x^6 + 2x^5 + 6x^3 is

Answers

Answer:

6

Step-by-step explanation:

As the highest power of x in the expression is 6, the degree is 6

Answer:

3x6+2x5+6x3−5

Standard Form

Step-by-step explanation:

HAVE A BLESS DAY :D

what is the sum for the 20th row and the 50th row

Answers

Answer:

Step-by-step explanation:

lkj

The algebraic expression for The quotient of 86 and x

Answers

Answer: 86 ÷ x or 86 / x

Step-by-step explanation:

A quotient is a number that results when one number is divided by another.

Therefore, in order to get the quotient, numbers or terms should process division which the final answer would be quotient.

In the process, [÷ ] or [/] the division signs shall be applied.

The quotient of 86 and x represents 86 divided by x, which the final result would be [tex]\boxed {86/x}[/tex]

Hope this helps!! :)

Please let me know if you have any questions

I believe the answer you are looking for is simply just 86÷x or 86/x.

cos a/4= -1/4
cos a= ??

Answers

Answer:

cos a / 4 = -1/4

--->cos a =- 1          

         

                                                                 

Emma jogs 2 miles along the beach in 1/3 of an hour. If she travels at a constant rate, how far will she jog in an hour?

Answers

Answer:

6

Step-by-step explanation:

you do 2 times 3 because you 2 miles is only 1/3 and you want do get the answer for an hour so 2/3 is 6

Answer:

6 miles

Step-by-step explanation:

If emma jogs at a constant rate she will travel 6 miles in 1 hour.

1/3 of an hour times 3 equals 1 hour and the 2 miles every 1/3 times 3 will be 6

hope it helps

What was Veronica's speed if she drove 475 miles in 6 1/4 hours?

Answers

Answer: 76 mph

Step-by-step explanation: speed is distance divided by time so 475 : 6.25 = 76 (mph)

Answer:

76 mph

Step-by-step explanation:

To find the answer, you would divide the miles she drove: 475 by the amount of time it took to get there: 6 1/4 hours. Convert the fraction to a decimal: 1/4= .25 so your problem would look like this...

475 / 6.25 = 76

why are there three zeros in the product of 6 times 103

Answers

Answer:

There’s not

Step-by-step explanation:

It’s 618

WILL GIVE BRAINLIEST TO WHOEVER CAN EXPLAIN:
True or False-in scientific notation when you move the decimal point left is it positive.

Answers

Answer:

false.

Step-by-step explanation:

just cause you move a decimal point to the left dosent mean its positive.

What does x equal?
2x + 5x – 4= 17


Answers

Answer:

x = 3

Step-by-step explanation:

2x + 5x - 4 = 17

Combine like terms

7x - 4 = 17

Add 4 to both sides

7x = 21

Solve for x

x = 21/7

x = 3
___________________
Hope this helps!

Love you.

- doomdabomb

Answer:

x = 3

Step-by-step explanation:

2x + 5x - 4 = 17      (Given)

7x - 4 = 17              (Simplify)

7x - 4 + 4 = 17 + 4  (Add 4 on both sides)

7x = 21                   (Simplify)

7x/7 = 21/7             (Divide 7 on both sides)

x = 3                      (Simplify)

Find a polynomial f(x) of degree 4 that has the following zeros. 2, 5, 0, -7

Answers

Answer:

Step-by-step explanation:

f(x) = (x - 2)(x - 5)x(x+ 7)

f(x) = (x^2 - 7x + 10)*x * (x + 7)

f(x) = x(x^3 - 39x + 70)

f(x) = x^4 - 39x^2 + 70x

To show that this is correct, I've made a graph with these points labeled. The graph is just around the x axis. The local maximums and minimums are just too large a value.

8w-2=12w+6 both answers

Answers

Answer:

your answer is -2

I hope it's helps you

Answer:

[tex]8w - 2 = 12w + 6 \\ 8w - 12w = 6 + 2 \\ - 4w = 8 \\ w = \frac{8}{ - 4} \\ w = - 2[/tex]

I hope it helped U

stay safe stay happy


6.55743... is an irrational number because it is an
a terminating decimal
a repeating decimal
a non-terminating and non-repeating decimal
a non-repeating decimal
a non-terminating decimal

Answers

Answer:

#3. a non-terminating and non-repeating

Step-by-step explanation:

6.55743... never ends and it never repeats either

Answer:

Not terminating but doesn't repeat either

Step-by-step explanation:

2f - 3(4f - 5g) + 5g + 7fg
Help pls how to solve it?

Answers

Step-by-step explanation:

2f - 3(4f - 5g) + 5g +7fg

multiply the 3 in

2f - 12f - 15g + 5g + 7fg

add or subtract common terms

-10f -10g +7fg will be your final answer

Answer:

Step-by-step explanation:

As the question did not say how to 'solve the question', I would assume it will be to simplify the equation, as there is no way of solving an expression without an answer.

Simplify:

2f - 3(4f-5g) + 5g +7fg

Get rid of the parentheses

2f - 12f - 15g + 5g +7fg

Simplify similar expressions

-10f - 10g + 7fg

Therefore, 2f - 3(4f-5g) + 5g +7fg simplified is equal to -10f - 10g + 7fg.

-5x²-2x³-4x Write the polynomial in standard form. Then identify the polynomial by the number of terms.

Answers

-2x^3-5x^2-4x
Cubic Trinomial

Answer:

-2x³ - 5x² - 4x and cubic

Step-by-step explanation:

→ Rearrange the polynomial so the highest power is at the start

-2x³ - 5x² - 4x

→ Now look at the highest power and determine the type of polynomial

Cubic

V=s^2h/3
Find the volume of a square pyramid If ‘s’ is increased by 30% and ‘h’ remains constant

Answers

Answer:

1.69s^2h/3.

Step-by-step explanation:

New s = 1.3s

New volume = (1.3s)^2h /3

= 1.69s^2h/3.

The speed of a falling object increases at a constant rate as time increases since the object was dropped. Which graph could represent the relationship between t, time in seconds, and s, speed in meters per second? A graph shows dropping time (seconds) labeled 1 to 10 on the horizontal axis and object speed (meters per second) on the vertical axis. A line increases from 0 to 10 seconds. A graph shows dropping time (seconds) labeled 1 to 10 on the horizontal axis and object speed (meters per second) on the vertical axis. A line decreases from 0 to 4 seconds. A graph shows dropping time (seconds) labeled 1 to 10 on the horizontal axis and object speed (meters per second) on the vertical axis. A line increases from 0 to 4.5 seconds. A graph shows dropping time (seconds) labeled 1 to 10 on the horizontal axis and object speed (meters per second) on the vertical axis. A line decreases from 1 to 9 seconds.

Answers

Answer:

Since the guy on top was so difficult the answer is A

Step-by-step explanation:

Edge 2021

Answer: Option A if your doing Edge.

Step-by-step explanation:

none

x-1/y-4/xy=-1
-3/y-12/xy=1

Answers

Answer:

1) x-1/y-4/xy=-1 Number one, the answer is in picture one.

2) -3/y-12/xy=1 Second, the answer is in picture two.

The table below shows Mandy's test scores what is the mean of her test scores
78 89 83 80 84
95 100 86 84 91

A 88 b 87 c 85 d 84

Answers

B. 87
78+89+83+80+84+95+100+86+84+91=870
870/10=87

Answer:

B,87

STEPS

78+89+83+80+84+95+100+86+84+91

10

870

10

A van moves with a constant speed of 116 km/h. How long will it take to travel a distance of 174 kilometers?

Answers

Answer:

1.5 hours / 90 minutes

Step-by-step explanation:

time = distance ÷ speed

if x and y are two overlapping subsets of a universal set U, write the relation between n(U),n(X U Y), and n(xūy).​

Answers

Answer:

the relation between them are

n(U) = n(X U Y) + n(xūy).

HELP ASAP!!!

The line (y-1)=(2/3)(x+1) contains point (a,-3).

What is the value of a? Show
your work.​

Answers

[tex]\\ \sf\Rrightarrow y-1=\dfrac{2}{3}(x+1)[/tex]

[tex]\\ \sf\Rrightarrow -3-1=\dfrac{2}{3}(a+1)[/tex]

[tex]\\ \sf\Rrightarrow -4=\dfrac{-2(a+1)}{3}[/tex]

[tex]\\ \sf\Rrightarrow -2(a+1)=-4(3)[/tex]

[tex]\\ \sf\Rrightarrow -2(a+1)=-12[/tex]

[tex]\\ \sf\Rrightarrow a+1=\dfrac{-12}{-2}[/tex]

[tex]\\ \sf\Rrightarrow a+1=6[/tex]

[tex]\\ \sf\Rrightarrow a=6-1[/tex]

[tex]\\ \sf\Rrightarrow a=5[/tex]

Jace has consumed 1,500 calories so far today. He has also burned off 400 calories at the gym. He would like to keep his daily calorie total to 1,900 calories per day. How many calories does he have left to consume for the day? Is 1,100 a viable solution to this problem?

Yes; 1,100 is less than 1,500.
No; 1,100 is more than the 400 he burned off at the gym.
No; 1,100 will cause him to exceed 1,900.
Yes; 1,100 is less than 1,900.

Answers

Answer:

Option 3

Step-by-step explanation:

If he burned off 400 calories from 1500 consumed calories, he has 1100 consumed calories.  1900 - 1100 = 800 calories left to consume.

While option 2 also says no, it's not the right reason.  

The calories left to consume for the day is 800 calories.

No, 1,100 will cause him to exceed 1,900.

Given,

Jace has consumed 1,500 calories so far today.

He has also burned off 400 calories at the gym.

He would like to keep his daily calorie total to 1,900 calories per day.

We need to find how many calories he has left to consume for the day.

And see which option is the best choice.

We have,

Calories consumed in a day = 1500

Burned calories = 400

Calories to keep in a day = 1900

Find the Calories left

= 1500 - 400

= 1100

Find how many calories need to be consumed to keep 1900 calories.

= Calories needed to keep in a day - calories remaining after burned

= 1900 - 1100

= 800

We see that the calories left to consume is 800 calories.

So 1100 is not a viable solution. It will cause him to exceed 1,900.

Thus,

The calories left to consume for the day is 800 calories.

No, 1,100 will cause him to exceed 1,900.

Learn more about finding the calories needed to consumed to keep daily calories here:

https://brainly.com/question/12862654

#SPJ2

The following shapes are similar. AB : IJ is 5:1 and the length of FE is 30. What is the length of NM?

Answers

Answer:

6

Step-by-step explanation:

5:1

30:6

multiply by 6 so its 6

Let w 1-5 and z=2+ 4i, Find z+w

Answers

Step-by-step explanation:

w+z=1-5+2+4¡

w+z=-2+4¡

and if you mean w= 1-5¡ , z= 2+4¡

z+w= 1-5¡+2+4¡

z+w= 3-¡

A polygon which is regular has interior angles measuring 150°. This polygon is called a:​

Answers

Answer:

Step-by-step explanation:

equilateral triangle

Answer:

Dodecagon (12 - sided polygon)

Step-by-step explanation:

Let it be n- sided polygon.

Sum of interior angles is 150n.

On the other hand we know the sum of interior angles of a regular n - sided polygon is calculated using the following formula:

S = 180(n - 2)

Compare the sums and solve for n:

180(n - 2) = 150n180n - 360 = 150n180n - 150n = 36030n = 360n = 360/30n = 12

12-sided polygon is called dodecagon

Solve for x and z (and find the total or a fraction or root of the equation)

4+x-3z*(7+3+6)=?

Answers

Answer:

Step-by-step explanation:

4+x-3*z(7+3+6)

4+x-3*z*16

4+x-3*16z

4+x-48z

x-48z+4

Four times the sum of three and a number

Answers

Answer:

4*3+X

IGNORE THIS JWINFWONDIWJFIENFIWKQPXK

3.
Find the number of real number solutions for the equation. x2 – 3x + 8 = 0

A. cannot be determined
B. 1
C. 2
D. 0

Answers

One

Is it 2x-3x+8=0,right?

Answer:

D. 0

Step-by-step explanation:

the answer to the equation is 0

3.  

Find the number of real number solutions for the equation. x2 – 3x + 8 = 0

cannot be determined

1

2

0

16 The cost for parking at a city airport is shown in the table.
Price for first day
$16.60
For every
2
day afterwards
$9
Yuri pays $124.60 to park his car.
How many days does he park his car for?
Show your working.

Answers

Answer:

He paid $9 twelve times

12× 2= 24 days

Step-by-step explanation:

1st day = $16.60

3rd day=$ 9 (i.e for every 2days afterwards)

5th day=$ 9

Yuri pays : 124.60

124.60-16.60

= $108.00

After paying the first day he paid 108.00

108.00÷ 9 = 12

He paid $9 twelve times

12× 2= 24 days

Other Questions
GIVING BRAIN-LIST TO THE CORRECT ANSWER!!!!!!!!______________________You can trust scientific information from advertisers because they are selling a product.True or false? What is a secondary source?A. A source of historical information that is not written down B. A source of historical information that has not been evaluated byan expertC. A source of historical information based on the interpretation ofother sources of historical evidenceD. A firsthand source of historical information Is this an effective summary of the story? Simplify.What is the answer? Arterionecrosis is the abnormal softening of the walls of an artery orarteries.a. Trueb. False The mass of an atom of element x is equivalent to the total mass of 7 hydrogen atoms Please put the following symptoms of the common cold in the order in which they usually appear. Runny nose, itchy throat, cough, chest congestion Solve for x please will mark brainliest Evaluate the expression Find the quotient. Round to the hundredths place.99.466 divided by 14.8 Select the best answer for this dialogue.Doctor: Hola, Sebastin.Young child Sebastin: Buenas tardes, doctor.Doctor: ________, Sebastin?Young child Sebastin: Estoy mal.Doctor: Lo siento.a.Cmo est b. Cmo estsc. Ms o menos d. Bien help pls its ez :)))))))))))))))) Describe the condition of sculpture in Nepal during Lichhavi period. 5. A female client in labor is receiving ceftaroline. She suddenly complains of trouble breathing, weakness, andnausea. The nurse should recognize that these signs are usually indicative of impending:Gebrows frA. Amniotic fluid embolismB. Pulmonary embolismC. AnaphylaxisD.Bronchospasm 8, 13, 18, 23 how to write out the sequence Nth term Help plz fast due 1159Write a paragraph of description on anything that comes to mind without using evaluative adjectives or adverbs. Two skaters stand on ice. one weights 250 N, and the other weights 500 N. They push against each other and move in opposite directions. Describe the momentum of each skater after they push away from each other. some one like Nepal??? What island is located at 5 52' 42" N, 162 4' 8" W? HELP PLEASE I NEED THIS ONE!!! If n/30 lies between 1/5 and 1/3, what are all the possible values of n if n is a whole number? Ill give brainliest